Minimize Samuel's fraction

Algebra Level 4

What is the minimum value of ( x 2 + x + 1 ) ( y 2 + y + 1 ) ( z 2 + z + 1 ) ( t 2 + t + 1 ) x y z t \left| \frac{(x^{2} + x + 1)(y^{2} + y + 1)(z^{2} + z + 1)(t^{2} + t + 1)}{xyzt} \right| as x , y , z , t x,y,z,t range over all non-zero real numbers?

This problem is posed by Samuel Q .


The answer is 1.

This section requires Javascript.
You are seeing this because something didn't load right. We suggest you, (a) try refreshing the page, (b) enabling javascript if it is disabled on your browser and, finally, (c) loading the non-javascript version of this page . We're sorry about the hassle.

7 solutions

Sujoy Roy
Oct 28, 2013

When x , y , z , t x, y, z, t all are negative real numbers (say x = p , y = q , z = r x=-p, y=-q, z=-r and t = s t=-s respectively), then the expression becomes ( p 2 p + 1 p ) ( q 2 q + 1 q ) ( r 2 r + 1 r ) ( s 2 s + 1 s ) = ( p + 1 p 1 ) ( q + 1 q 1 ) ( r + 1 r 1 ) ( s + 1 s 1 ) ( 2 1 ) ( 2 1 ) ( 2 1 ) ( 2 1 ) = 1 |(\frac{p^2-p+1}{p})(\frac{q^2-q+1}{q})(\frac{r^2-r+1}{r})(\frac{s^2-s+1}{s})| = |(p+\frac{1}{p}-1)(q+\frac{1}{q}-1)(r+\frac{1}{r}-1)(s+\frac{1}{s}-1)| \ge |(2-1)(2-1)(2-1)(2-1)| = 1 as k + 1 k 2 k+\frac{1}{k} \ge 2 for positive real number k k . So minimum value is 1.

What if not all of them are negative real numbers?

Calvin Lin Staff - 7 years, 7 months ago

If any one of x , y , z , t x, y, z, t is positive real that contributes at least 3, as k 2 + k + 1 k = k + 1 k + 1 3 |\frac{k^2+k+1}{k}|=|k+\frac{1}{k}+1| \ge 3 . If any one of x , y , z , t x, y, z, t is negative real that contributes at least 1. The expression is minimum when all x , y , z , t x, y, z, t contribute 1.

sujoy roy - 7 years, 7 months ago

Log in to reply

Great! This is a major gap in your writeup which you have now filled in.

Calvin Lin Staff - 7 years, 7 months ago
Sandeep Bhardwaj
Sep 18, 2014

We can rewrite the given equations as:

[ x + 1 + 1 x ] [ y + 1 + 1 y ] [ z + 1 + 1 z ] [ t + 1 + 1 t ] \large [x+1+\frac{1}{x}][y+1+\frac{1}{y}][z+1+\frac{1}{z}][t+1+\frac{1}{t}]

[ x + 1 + 1 x ] 3 \large [x+1+\frac{1}{x}] \geq 3 if x is positive

[ x + 1 + 1 x ] 1 \large [x+1+\frac{1}{x}] \leq -1 if x is negative

We need minimum value , thats why we will consider the second case. And modulus is applied over the whole expression, so we will take every term (in product) equal to -1.

So min. value = 1 = 1 \large =\boxed{|-1|=1}

I too did the same

Mehul Chaturvedi - 6 years, 5 months ago
Eric Wu
Oct 31, 2013

( x 2 + x + 1 ) ( y 2 + y + 1 ) ( z 2 + z + 1 ) ( t 2 + t + 1 ) x y z t |\frac{(x^2+x+1)(y^2+y+1)(z^2+z+1)(t^2+t+1)}{xyzt}|

Since all four variables are symmetric, we can write this as x 2 + x + 1 x 4 |\frac{x^2+x+1}{x}|^4

x 2 + x + 1 x 4 = x + 1 + 1 x 4 |\frac{x^2+x+1}{x}|^4= |x+1+\frac{1}{x}|^4

x + 1 + 1 x 4 1 4 |x+1+\frac{1}{x}|^4\ge1^4 through use of the AM-GM inequality

Thus, our answer is one.

If you apply AM-GM on last equation you get ( x + 1 + 1 x ) 4 ( 3 1 3 ) 4 = 3 4 = 81 (x + 1 + \frac{1}{x})^4 \ge (3\sqrt[3]{1})^4 = 3^4 = 81 which is false. You can apply AM-GM if the numbers are positive reals.

Jordi Bosch - 6 years, 7 months ago
Sameer Prasai
Oct 27, 2013

The expression is the product of 4 symmetric expressions. We see that each such expression can be written as (1+x+1/x) which defines a curve with certain minima. Since this minima is same for all 4 expressions, the value of x, y, z & t is same for the minimum value. Thus the expression can be written as (1 + x + 1/x)^4. By calculating the minima of the expression, we find 1 & -1 as the optimum values. It is obvious from the expression that a negative value of x yields lesser value of expression. So putting x = -1, the value of expression becomes (-1)^4 = 1.

You have to be careful with your argument, it need not be true that f g fg is minimal when f f and g g are both minimal (or equal), especially if they are allowed to be negative.

For example, if f f and g g are allowed to be 2 or -2, then the minimum value of f g fg is 4 -4 , which occurs at 2 × ( 2 ) 2 \times (-2) or ( 2 ) × 2 (-2) \times 2 . It certainly doesn't occur at ( 2 ) × ( 2 ) = 4 (-2) \times (-2) = 4 .

Calvin Lin Staff - 7 years, 7 months ago

Log in to reply

I see, but what about the absolute value of fg? It is minimum when the magnitude of both f and g is minimum, i.e. f=g=0. I considered the exact same argument because the expression was the absolute value. So in this case, the minimum value depends solely on the magnitude. In such situations, I believe the value of magnitude of all terms in the expression must be minimum and equal. I actually solved this question graphically, without using minima but it was difficult to explain that in solution. So I explained a different method.

Sameer Prasai - 7 years, 7 months ago

Log in to reply

Yes, when we are restricted to just positive (non-negative) values, then the minimum of f g fg occurs at the minimum of f f multiplies by the minimum of g g . You needed to have stated this fact, in order for your solution to be complete.

How would you demonstrate that this (seemingly obvious) fact is true? Why doesn't it work when we allow for negative values (as demonstrated by my initial example) ?

Calvin Lin Staff - 7 years, 7 months ago
Fox To-ong
Jan 14, 2015

simplifying the given data, the result is the absolute value of -1 = 1

Sajan Kapil
Feb 2, 2014

minimum of (x+1/x) is at x=-1; hence x+1/x+1=-1; so -1* -1 - 1 -1=1

David Popovic
Oct 28, 2013

Plugging in x = y = z = t = 1 x=y=z=t=-1 we see that 1 1 is possible.

0 0 is not possible, because q 2 + q + 1 = 0 q^2+q+1=0 does not have a real root.

How would you show that 1 is indeed the minimum, instead of merely showing that it can be achieved?

Calvin Lin Staff - 7 years, 7 months ago

0 pending reports

×

Problem Loading...

Note Loading...

Set Loading...